An uncountable set has uncountably many co-countable subsets containing an arbitrary point Announcing the arrival of Valued Associate #679: Cesar Manara Planned maintenance scheduled April 23, 2019 at 00:00UTC (8:00pm US/Eastern)Uncountable minus countable set is uncountableCountable subset of a uncountable setCountable subsets of an uncountable setAre there countably or uncountably many infinite subsets of the positive even integers?A countable and an uncountable setIs there an infinite countable $sigma$-algebra on an uncountable setEvery infinite set has an infinite countable subset?Can subset of a countable set be uncountable?Uncountable Ordered Set Isomorphic to to Its Uncountable SubsetsIf $A$ is infinite, then $mathcalP(A)$ is uncountable

How does the math work when buying airline miles?

Is grep documentation about ignoring case wrong, since it doesn't ignore case in filenames?

Central Vacuuming: Is it worth it, and how does it compare to normal vacuuming?

Putting class ranking in CV, but against dept guidelines

An adverb for when you're not exaggerating

How do I use the new nonlinear finite element in Mathematica 12 for this equation?

Illegal assignment from sObject to Id

How does light 'choose' between wave and particle behaviour?

Why is Nikon 1.4g better when Nikon 1.8g is sharper?

Is there a kind of relay that only consumes power when switching?

How to react to hostile behavior from a senior developer?

What are the diatonic extended chords of C major?

Taylor expansion of ln(1-x)

Is it fair for a professor to grade us on the possession of past papers?

Is it a good idea to use CNN to classify 1D signal?

What is a fractional matching?

Why do early math courses focus on the cross sections of a cone and not on other 3D objects?

Why should I vote and accept answers?

Can a new player join a group only when a new campaign starts?

Is a ledger board required if the side of my house is wood?

Amount of permutations on an NxNxN Rubik's Cube

Should I use a zero-interest credit card for a large one-time purchase?

Can the Great Weapon Master feat's damage bonus and accuracy penalty apply to attacks from the Spiritual Weapon spell?

Is it possible for SQL statements to execute concurrently within a single session in SQL Server?



An uncountable set has uncountably many co-countable subsets containing an arbitrary point



Announcing the arrival of Valued Associate #679: Cesar Manara
Planned maintenance scheduled April 23, 2019 at 00:00UTC (8:00pm US/Eastern)Uncountable minus countable set is uncountableCountable subset of a uncountable setCountable subsets of an uncountable setAre there countably or uncountably many infinite subsets of the positive even integers?A countable and an uncountable setIs there an infinite countable $sigma$-algebra on an uncountable setEvery infinite set has an infinite countable subset?Can subset of a countable set be uncountable?Uncountable Ordered Set Isomorphic to to Its Uncountable SubsetsIf $A$ is infinite, then $mathcalP(A)$ is uncountable










0












$begingroup$


This theorem seems "obvious" to me, but I want to check my logic since I am un-familiar with un-countably infinite sets, and I know these can give rise to non-intuitive results. Any comments welcome.



Definitions



The cardinality of a set $A$ will be denoted $|A|$. If $|A|=|mathbbN|$, $A$ will be called countable or countably infinite and this will be denoted $|A|=aleph_0$. If $A$ is infinite and $|A|nealeph_0$, $A$ will be called uncountable or uncountably infinite.



Claim



If $A$ is un-countable then for any $ain A$, $A$ contains an uncountable number of disjoint co-countable sets that contain $a$. More formally;




For any $ain A$ where $|A|not=aleph_0$ there exist subsets
$B_betasubset A$, $betain I_beta$ for an index set
$I_beta$, $|I_beta|not=aleph_0$, such that for all $betain I_beta$
: $ain B_beta$ and $|A-B_beta|=aleph_0$.
Furthermore for any $betanot=beta'$, $B_betacap B_beta'=emptyset$




Proof



Without loss of generality let $I_beta=mathbbR$ and choose $betain I_beta$. Choose $a_beta,1in A$, $a_beta,1not=a$. Since $|A-a_beta,1|not=aleph_0$ there exists $a_beta,2inA-a_beta,1$, $a_beta,2not=a$. Since $|A-a_beta,1,a_beta,2|not=aleph_0$ there exists $a_beta,3inA-a_beta,1,a_beta,2$, $a_beta,3not=a$. Continuing in this way $n_beta$ times, $n_betainmathbbN$, we can construct a set $B_beta=A-a_beta,1,a_beta,2,...,a_beta,n_beta$ that satisfies $ain B_betasubset A$ and $|A-B_beta|=|a_beta,1,a_beta,2,...,a_beta,n_beta|=aleph_0$ so that $B_beta$ is a co-countable subset of $A$ containing $a$.



Choose another $beta'in I_beta$, $betanot=beta'$. Since $|B_beta|not=aleph_0$ there exists $a_beta',1in B_beta$, $a_beta',1not=a$. Since $|B_beta-a_beta',1|not=aleph_0$ there exists $a_beta',2in B_beta-a_beta',1$, $a_beta',2not=a$. Since $|B_beta-a_beta',1,a_beta',2|not=aleph_0$ there exists $a_beta',3in B_beta-a_beta',1,a_beta',2$, $a_beta',3not=a$. Continuing in this way $n_beta'$ times, $n_beta'inmathbbN$, we can construct a set $B_beta'=B_beta-a_beta',1,a_beta',2,...,a_beta',n_beta'$ that satisfies $ain B_beta'subset A$ and $|A-B_beta'|=|a_beta,1,a_beta,2,...,a_beta,n_betacupa_beta,1,a_beta,2,...,a_beta',n_beta'|=aleph_0+aleph_0=aleph_0$ so that $B_beta'$ is a co-countable subset of $A$ containing $a$.



Since $B_betacap B_beta'=emptyset$ and since we can choose un-countably many $betain I_beta$ the result is proved $hspace5ptblacksquare$










share|cite|improve this question











$endgroup$







  • 3




    $begingroup$
    Your definition of "uncountably infinite" is not standard. Usually, it is an infinite set $S$ such that $|S|neq aleph_0.$ There are a lot of uncountably infinite cardinals other than $|mathbb R.|$
    $endgroup$
    – Thomas Andrews
    Mar 27 at 17:31







  • 1




    $begingroup$
    No, you are wrong, @AndrésE.Caicedo Given $ain A$ take a countable subset $Csubseteq A$ with $anotin C.$ Then take $Asetminus Tmid Tsubset C, text and T infinite$. There are uncountably many (countably) infinite subsets $T$ of $C$, and for each $ain A-T,$ and $A-T$ is co-countable.
    $endgroup$
    – Thomas Andrews
    Mar 27 at 18:03







  • 1




    $begingroup$
    @Thomas These sets are not disjoint.
    $endgroup$
    – Andrés E. Caicedo
    Mar 27 at 18:05






  • 1




    $begingroup$
    @dandar Without the requirement of disjointness, of course the claim holds, and it is straightforward. Thomas sketched in comments what to do. I do not see uncountably many sets produced using procedures along the lines your comment suggests. If besides disjointness we also ignore the requirement of having uncountably many such sets, then yes, your comment gives a possible approach.
    $endgroup$
    – Andrés E. Caicedo
    Mar 28 at 16:27







  • 1




    $begingroup$
    @dandar Yes, that's the argument.
    $endgroup$
    – Andrés E. Caicedo
    Mar 28 at 18:33















0












$begingroup$


This theorem seems "obvious" to me, but I want to check my logic since I am un-familiar with un-countably infinite sets, and I know these can give rise to non-intuitive results. Any comments welcome.



Definitions



The cardinality of a set $A$ will be denoted $|A|$. If $|A|=|mathbbN|$, $A$ will be called countable or countably infinite and this will be denoted $|A|=aleph_0$. If $A$ is infinite and $|A|nealeph_0$, $A$ will be called uncountable or uncountably infinite.



Claim



If $A$ is un-countable then for any $ain A$, $A$ contains an uncountable number of disjoint co-countable sets that contain $a$. More formally;




For any $ain A$ where $|A|not=aleph_0$ there exist subsets
$B_betasubset A$, $betain I_beta$ for an index set
$I_beta$, $|I_beta|not=aleph_0$, such that for all $betain I_beta$
: $ain B_beta$ and $|A-B_beta|=aleph_0$.
Furthermore for any $betanot=beta'$, $B_betacap B_beta'=emptyset$




Proof



Without loss of generality let $I_beta=mathbbR$ and choose $betain I_beta$. Choose $a_beta,1in A$, $a_beta,1not=a$. Since $|A-a_beta,1|not=aleph_0$ there exists $a_beta,2inA-a_beta,1$, $a_beta,2not=a$. Since $|A-a_beta,1,a_beta,2|not=aleph_0$ there exists $a_beta,3inA-a_beta,1,a_beta,2$, $a_beta,3not=a$. Continuing in this way $n_beta$ times, $n_betainmathbbN$, we can construct a set $B_beta=A-a_beta,1,a_beta,2,...,a_beta,n_beta$ that satisfies $ain B_betasubset A$ and $|A-B_beta|=|a_beta,1,a_beta,2,...,a_beta,n_beta|=aleph_0$ so that $B_beta$ is a co-countable subset of $A$ containing $a$.



Choose another $beta'in I_beta$, $betanot=beta'$. Since $|B_beta|not=aleph_0$ there exists $a_beta',1in B_beta$, $a_beta',1not=a$. Since $|B_beta-a_beta',1|not=aleph_0$ there exists $a_beta',2in B_beta-a_beta',1$, $a_beta',2not=a$. Since $|B_beta-a_beta',1,a_beta',2|not=aleph_0$ there exists $a_beta',3in B_beta-a_beta',1,a_beta',2$, $a_beta',3not=a$. Continuing in this way $n_beta'$ times, $n_beta'inmathbbN$, we can construct a set $B_beta'=B_beta-a_beta',1,a_beta',2,...,a_beta',n_beta'$ that satisfies $ain B_beta'subset A$ and $|A-B_beta'|=|a_beta,1,a_beta,2,...,a_beta,n_betacupa_beta,1,a_beta,2,...,a_beta',n_beta'|=aleph_0+aleph_0=aleph_0$ so that $B_beta'$ is a co-countable subset of $A$ containing $a$.



Since $B_betacap B_beta'=emptyset$ and since we can choose un-countably many $betain I_beta$ the result is proved $hspace5ptblacksquare$










share|cite|improve this question











$endgroup$







  • 3




    $begingroup$
    Your definition of "uncountably infinite" is not standard. Usually, it is an infinite set $S$ such that $|S|neq aleph_0.$ There are a lot of uncountably infinite cardinals other than $|mathbb R.|$
    $endgroup$
    – Thomas Andrews
    Mar 27 at 17:31







  • 1




    $begingroup$
    No, you are wrong, @AndrésE.Caicedo Given $ain A$ take a countable subset $Csubseteq A$ with $anotin C.$ Then take $Asetminus Tmid Tsubset C, text and T infinite$. There are uncountably many (countably) infinite subsets $T$ of $C$, and for each $ain A-T,$ and $A-T$ is co-countable.
    $endgroup$
    – Thomas Andrews
    Mar 27 at 18:03







  • 1




    $begingroup$
    @Thomas These sets are not disjoint.
    $endgroup$
    – Andrés E. Caicedo
    Mar 27 at 18:05






  • 1




    $begingroup$
    @dandar Without the requirement of disjointness, of course the claim holds, and it is straightforward. Thomas sketched in comments what to do. I do not see uncountably many sets produced using procedures along the lines your comment suggests. If besides disjointness we also ignore the requirement of having uncountably many such sets, then yes, your comment gives a possible approach.
    $endgroup$
    – Andrés E. Caicedo
    Mar 28 at 16:27







  • 1




    $begingroup$
    @dandar Yes, that's the argument.
    $endgroup$
    – Andrés E. Caicedo
    Mar 28 at 18:33













0












0








0





$begingroup$


This theorem seems "obvious" to me, but I want to check my logic since I am un-familiar with un-countably infinite sets, and I know these can give rise to non-intuitive results. Any comments welcome.



Definitions



The cardinality of a set $A$ will be denoted $|A|$. If $|A|=|mathbbN|$, $A$ will be called countable or countably infinite and this will be denoted $|A|=aleph_0$. If $A$ is infinite and $|A|nealeph_0$, $A$ will be called uncountable or uncountably infinite.



Claim



If $A$ is un-countable then for any $ain A$, $A$ contains an uncountable number of disjoint co-countable sets that contain $a$. More formally;




For any $ain A$ where $|A|not=aleph_0$ there exist subsets
$B_betasubset A$, $betain I_beta$ for an index set
$I_beta$, $|I_beta|not=aleph_0$, such that for all $betain I_beta$
: $ain B_beta$ and $|A-B_beta|=aleph_0$.
Furthermore for any $betanot=beta'$, $B_betacap B_beta'=emptyset$




Proof



Without loss of generality let $I_beta=mathbbR$ and choose $betain I_beta$. Choose $a_beta,1in A$, $a_beta,1not=a$. Since $|A-a_beta,1|not=aleph_0$ there exists $a_beta,2inA-a_beta,1$, $a_beta,2not=a$. Since $|A-a_beta,1,a_beta,2|not=aleph_0$ there exists $a_beta,3inA-a_beta,1,a_beta,2$, $a_beta,3not=a$. Continuing in this way $n_beta$ times, $n_betainmathbbN$, we can construct a set $B_beta=A-a_beta,1,a_beta,2,...,a_beta,n_beta$ that satisfies $ain B_betasubset A$ and $|A-B_beta|=|a_beta,1,a_beta,2,...,a_beta,n_beta|=aleph_0$ so that $B_beta$ is a co-countable subset of $A$ containing $a$.



Choose another $beta'in I_beta$, $betanot=beta'$. Since $|B_beta|not=aleph_0$ there exists $a_beta',1in B_beta$, $a_beta',1not=a$. Since $|B_beta-a_beta',1|not=aleph_0$ there exists $a_beta',2in B_beta-a_beta',1$, $a_beta',2not=a$. Since $|B_beta-a_beta',1,a_beta',2|not=aleph_0$ there exists $a_beta',3in B_beta-a_beta',1,a_beta',2$, $a_beta',3not=a$. Continuing in this way $n_beta'$ times, $n_beta'inmathbbN$, we can construct a set $B_beta'=B_beta-a_beta',1,a_beta',2,...,a_beta',n_beta'$ that satisfies $ain B_beta'subset A$ and $|A-B_beta'|=|a_beta,1,a_beta,2,...,a_beta,n_betacupa_beta,1,a_beta,2,...,a_beta',n_beta'|=aleph_0+aleph_0=aleph_0$ so that $B_beta'$ is a co-countable subset of $A$ containing $a$.



Since $B_betacap B_beta'=emptyset$ and since we can choose un-countably many $betain I_beta$ the result is proved $hspace5ptblacksquare$










share|cite|improve this question











$endgroup$




This theorem seems "obvious" to me, but I want to check my logic since I am un-familiar with un-countably infinite sets, and I know these can give rise to non-intuitive results. Any comments welcome.



Definitions



The cardinality of a set $A$ will be denoted $|A|$. If $|A|=|mathbbN|$, $A$ will be called countable or countably infinite and this will be denoted $|A|=aleph_0$. If $A$ is infinite and $|A|nealeph_0$, $A$ will be called uncountable or uncountably infinite.



Claim



If $A$ is un-countable then for any $ain A$, $A$ contains an uncountable number of disjoint co-countable sets that contain $a$. More formally;




For any $ain A$ where $|A|not=aleph_0$ there exist subsets
$B_betasubset A$, $betain I_beta$ for an index set
$I_beta$, $|I_beta|not=aleph_0$, such that for all $betain I_beta$
: $ain B_beta$ and $|A-B_beta|=aleph_0$.
Furthermore for any $betanot=beta'$, $B_betacap B_beta'=emptyset$




Proof



Without loss of generality let $I_beta=mathbbR$ and choose $betain I_beta$. Choose $a_beta,1in A$, $a_beta,1not=a$. Since $|A-a_beta,1|not=aleph_0$ there exists $a_beta,2inA-a_beta,1$, $a_beta,2not=a$. Since $|A-a_beta,1,a_beta,2|not=aleph_0$ there exists $a_beta,3inA-a_beta,1,a_beta,2$, $a_beta,3not=a$. Continuing in this way $n_beta$ times, $n_betainmathbbN$, we can construct a set $B_beta=A-a_beta,1,a_beta,2,...,a_beta,n_beta$ that satisfies $ain B_betasubset A$ and $|A-B_beta|=|a_beta,1,a_beta,2,...,a_beta,n_beta|=aleph_0$ so that $B_beta$ is a co-countable subset of $A$ containing $a$.



Choose another $beta'in I_beta$, $betanot=beta'$. Since $|B_beta|not=aleph_0$ there exists $a_beta',1in B_beta$, $a_beta',1not=a$. Since $|B_beta-a_beta',1|not=aleph_0$ there exists $a_beta',2in B_beta-a_beta',1$, $a_beta',2not=a$. Since $|B_beta-a_beta',1,a_beta',2|not=aleph_0$ there exists $a_beta',3in B_beta-a_beta',1,a_beta',2$, $a_beta',3not=a$. Continuing in this way $n_beta'$ times, $n_beta'inmathbbN$, we can construct a set $B_beta'=B_beta-a_beta',1,a_beta',2,...,a_beta',n_beta'$ that satisfies $ain B_beta'subset A$ and $|A-B_beta'|=|a_beta,1,a_beta,2,...,a_beta,n_betacupa_beta,1,a_beta,2,...,a_beta',n_beta'|=aleph_0+aleph_0=aleph_0$ so that $B_beta'$ is a co-countable subset of $A$ containing $a$.



Since $B_betacap B_beta'=emptyset$ and since we can choose un-countably many $betain I_beta$ the result is proved $hspace5ptblacksquare$







elementary-set-theory cardinals






share|cite|improve this question















share|cite|improve this question













share|cite|improve this question




share|cite|improve this question








edited Mar 27 at 17:57









Andrés E. Caicedo

66.1k8160252




66.1k8160252










asked Mar 27 at 17:28









dandardandar

415616




415616







  • 3




    $begingroup$
    Your definition of "uncountably infinite" is not standard. Usually, it is an infinite set $S$ such that $|S|neq aleph_0.$ There are a lot of uncountably infinite cardinals other than $|mathbb R.|$
    $endgroup$
    – Thomas Andrews
    Mar 27 at 17:31







  • 1




    $begingroup$
    No, you are wrong, @AndrésE.Caicedo Given $ain A$ take a countable subset $Csubseteq A$ with $anotin C.$ Then take $Asetminus Tmid Tsubset C, text and T infinite$. There are uncountably many (countably) infinite subsets $T$ of $C$, and for each $ain A-T,$ and $A-T$ is co-countable.
    $endgroup$
    – Thomas Andrews
    Mar 27 at 18:03







  • 1




    $begingroup$
    @Thomas These sets are not disjoint.
    $endgroup$
    – Andrés E. Caicedo
    Mar 27 at 18:05






  • 1




    $begingroup$
    @dandar Without the requirement of disjointness, of course the claim holds, and it is straightforward. Thomas sketched in comments what to do. I do not see uncountably many sets produced using procedures along the lines your comment suggests. If besides disjointness we also ignore the requirement of having uncountably many such sets, then yes, your comment gives a possible approach.
    $endgroup$
    – Andrés E. Caicedo
    Mar 28 at 16:27







  • 1




    $begingroup$
    @dandar Yes, that's the argument.
    $endgroup$
    – Andrés E. Caicedo
    Mar 28 at 18:33












  • 3




    $begingroup$
    Your definition of "uncountably infinite" is not standard. Usually, it is an infinite set $S$ such that $|S|neq aleph_0.$ There are a lot of uncountably infinite cardinals other than $|mathbb R.|$
    $endgroup$
    – Thomas Andrews
    Mar 27 at 17:31







  • 1




    $begingroup$
    No, you are wrong, @AndrésE.Caicedo Given $ain A$ take a countable subset $Csubseteq A$ with $anotin C.$ Then take $Asetminus Tmid Tsubset C, text and T infinite$. There are uncountably many (countably) infinite subsets $T$ of $C$, and for each $ain A-T,$ and $A-T$ is co-countable.
    $endgroup$
    – Thomas Andrews
    Mar 27 at 18:03







  • 1




    $begingroup$
    @Thomas These sets are not disjoint.
    $endgroup$
    – Andrés E. Caicedo
    Mar 27 at 18:05






  • 1




    $begingroup$
    @dandar Without the requirement of disjointness, of course the claim holds, and it is straightforward. Thomas sketched in comments what to do. I do not see uncountably many sets produced using procedures along the lines your comment suggests. If besides disjointness we also ignore the requirement of having uncountably many such sets, then yes, your comment gives a possible approach.
    $endgroup$
    – Andrés E. Caicedo
    Mar 28 at 16:27







  • 1




    $begingroup$
    @dandar Yes, that's the argument.
    $endgroup$
    – Andrés E. Caicedo
    Mar 28 at 18:33







3




3




$begingroup$
Your definition of "uncountably infinite" is not standard. Usually, it is an infinite set $S$ such that $|S|neq aleph_0.$ There are a lot of uncountably infinite cardinals other than $|mathbb R.|$
$endgroup$
– Thomas Andrews
Mar 27 at 17:31





$begingroup$
Your definition of "uncountably infinite" is not standard. Usually, it is an infinite set $S$ such that $|S|neq aleph_0.$ There are a lot of uncountably infinite cardinals other than $|mathbb R.|$
$endgroup$
– Thomas Andrews
Mar 27 at 17:31





1




1




$begingroup$
No, you are wrong, @AndrésE.Caicedo Given $ain A$ take a countable subset $Csubseteq A$ with $anotin C.$ Then take $Asetminus Tmid Tsubset C, text and T infinite$. There are uncountably many (countably) infinite subsets $T$ of $C$, and for each $ain A-T,$ and $A-T$ is co-countable.
$endgroup$
– Thomas Andrews
Mar 27 at 18:03





$begingroup$
No, you are wrong, @AndrésE.Caicedo Given $ain A$ take a countable subset $Csubseteq A$ with $anotin C.$ Then take $Asetminus Tmid Tsubset C, text and T infinite$. There are uncountably many (countably) infinite subsets $T$ of $C$, and for each $ain A-T,$ and $A-T$ is co-countable.
$endgroup$
– Thomas Andrews
Mar 27 at 18:03





1




1




$begingroup$
@Thomas These sets are not disjoint.
$endgroup$
– Andrés E. Caicedo
Mar 27 at 18:05




$begingroup$
@Thomas These sets are not disjoint.
$endgroup$
– Andrés E. Caicedo
Mar 27 at 18:05




1




1




$begingroup$
@dandar Without the requirement of disjointness, of course the claim holds, and it is straightforward. Thomas sketched in comments what to do. I do not see uncountably many sets produced using procedures along the lines your comment suggests. If besides disjointness we also ignore the requirement of having uncountably many such sets, then yes, your comment gives a possible approach.
$endgroup$
– Andrés E. Caicedo
Mar 28 at 16:27





$begingroup$
@dandar Without the requirement of disjointness, of course the claim holds, and it is straightforward. Thomas sketched in comments what to do. I do not see uncountably many sets produced using procedures along the lines your comment suggests. If besides disjointness we also ignore the requirement of having uncountably many such sets, then yes, your comment gives a possible approach.
$endgroup$
– Andrés E. Caicedo
Mar 28 at 16:27





1




1




$begingroup$
@dandar Yes, that's the argument.
$endgroup$
– Andrés E. Caicedo
Mar 28 at 18:33




$begingroup$
@dandar Yes, that's the argument.
$endgroup$
– Andrés E. Caicedo
Mar 28 at 18:33










0






active

oldest

votes












Your Answer








StackExchange.ready(function()
var channelOptions =
tags: "".split(" "),
id: "69"
;
initTagRenderer("".split(" "), "".split(" "), channelOptions);

StackExchange.using("externalEditor", function()
// Have to fire editor after snippets, if snippets enabled
if (StackExchange.settings.snippets.snippetsEnabled)
StackExchange.using("snippets", function()
createEditor();
);

else
createEditor();

);

function createEditor()
StackExchange.prepareEditor(
heartbeatType: 'answer',
autoActivateHeartbeat: false,
convertImagesToLinks: true,
noModals: true,
showLowRepImageUploadWarning: true,
reputationToPostImages: 10,
bindNavPrevention: true,
postfix: "",
imageUploader:
brandingHtml: "Powered by u003ca class="icon-imgur-white" href="https://imgur.com/"u003eu003c/au003e",
contentPolicyHtml: "User contributions licensed under u003ca href="https://creativecommons.org/licenses/by-sa/3.0/"u003ecc by-sa 3.0 with attribution requiredu003c/au003e u003ca href="https://stackoverflow.com/legal/content-policy"u003e(content policy)u003c/au003e",
allowUrls: true
,
noCode: true, onDemand: true,
discardSelector: ".discard-answer"
,immediatelyShowMarkdownHelp:true
);



);













draft saved

draft discarded


















StackExchange.ready(
function ()
StackExchange.openid.initPostLogin('.new-post-login', 'https%3a%2f%2fmath.stackexchange.com%2fquestions%2f3164824%2fan-uncountable-set-has-uncountably-many-co-countable-subsets-containing-an-arbit%23new-answer', 'question_page');

);

Post as a guest















Required, but never shown

























0






active

oldest

votes








0






active

oldest

votes









active

oldest

votes






active

oldest

votes















draft saved

draft discarded
















































Thanks for contributing an answer to Mathematics Stack Exchange!


  • Please be sure to answer the question. Provide details and share your research!

But avoid


  • Asking for help, clarification, or responding to other answers.

  • Making statements based on opinion; back them up with references or personal experience.

Use MathJax to format equations. MathJax reference.


To learn more, see our tips on writing great answers.




draft saved


draft discarded














StackExchange.ready(
function ()
StackExchange.openid.initPostLogin('.new-post-login', 'https%3a%2f%2fmath.stackexchange.com%2fquestions%2f3164824%2fan-uncountable-set-has-uncountably-many-co-countable-subsets-containing-an-arbit%23new-answer', 'question_page');

);

Post as a guest















Required, but never shown





















































Required, but never shown














Required, but never shown












Required, but never shown







Required, but never shown

































Required, but never shown














Required, but never shown












Required, but never shown







Required, but never shown







Popular posts from this blog

Lowndes Grove History Architecture References Navigation menu32°48′6″N 79°57′58″W / 32.80167°N 79.96611°W / 32.80167; -79.9661132°48′6″N 79°57′58″W / 32.80167°N 79.96611°W / 32.80167; -79.9661178002500"National Register Information System"Historic houses of South Carolina"Lowndes Grove""+32° 48' 6.00", −79° 57' 58.00""Lowndes Grove, Charleston County (260 St. Margaret St., Charleston)""Lowndes Grove"The Charleston ExpositionIt Happened in South Carolina"Lowndes Grove (House), Saint Margaret Street & Sixth Avenue, Charleston, Charleston County, SC(Photographs)"Plantations of the Carolina Low Countrye

random experiment with two different functions on unit interval Announcing the arrival of Valued Associate #679: Cesar Manara Planned maintenance scheduled April 23, 2019 at 00:00UTC (8:00pm US/Eastern)Random variable and probability space notionsRandom Walk with EdgesFinding functions where the increase over a random interval is Poisson distributedNumber of days until dayCan an observed event in fact be of zero probability?Unit random processmodels of coins and uniform distributionHow to get the number of successes given $n$ trials , probability $P$ and a random variable $X$Absorbing Markov chain in a computer. Is “almost every” turned into always convergence in computer executions?Stopped random walk is not uniformly integrable

How should I support this large drywall patch? Planned maintenance scheduled April 23, 2019 at 00:00UTC (8:00pm US/Eastern) Announcing the arrival of Valued Associate #679: Cesar Manara Unicorn Meta Zoo #1: Why another podcast?How do I cover large gaps in drywall?How do I keep drywall around a patch from crumbling?Can I glue a second layer of drywall?How to patch long strip on drywall?Large drywall patch: how to avoid bulging seams?Drywall Mesh Patch vs. Bulge? To remove or not to remove?How to fix this drywall job?Prep drywall before backsplashWhat's the best way to fix this horrible drywall patch job?Drywall patching using 3M Patch Plus Primer